2. Fiona is studying how income taxes impact various families and their finances. She creates a table with various amounts of taxes owed and estimates


that this represents 9% of each family's gross income.


Solve for the gross income for each family based off of their taxes owed.

Answers

Answer 1

The equation is Gross Income = Taxes Owed / 0.09.

To solve for the gross income for each family based on their taxes owed and the fact that this represents 9% of each family's gross income, follow these steps:

1. Write the equation: Taxes Owed = 0.09 * Gross Income
2. Rearrange the equation to solve for Gross Income: Gross Income = Taxes Owed / 0.09
3. Substitute the Taxes Owed value for each family into the equation and calculate their Gross Income.

For each family, input their taxes owed into the equation and you will find their gross income. Remember, the equation is Gross Income = Taxes Owed / 0.09.

To learn more about gross income

https://brainly.com/question/31269324

#SPJ11


Related Questions

use the given information to solve the triangle
C=135° C = 45₁ B = 10°
4)
5) A = 26°₁ a = 10₁ 6=4
6) A = 60°, a = 9₁ c = 10
7) A=150° C = 20° a = 200
8) A = 24.3°, C = 54.6°₁ C = 2.68
9) A = 83° 20′, C = 54.6°₁ c 18,1

Answers

The law of sines is solved and the triangle is given by the following relation

Given data ,

From the law of sines , we get

a / sin A = b / sin B = c / sin C

a)

C = 135° C = 45₁ B = 10°

So , the measure of triangle is

A/ ( 180 - 35 - 10 ) = A / 35

And , a/ ( sin 135/35 ) = sin 35 / a

On simplifying , we get

a = 36.50

Hence , the law of sines is solved

To learn more about law of sines click :

https://brainly.com/question/13098194

#SPJ1

How much must be deposited today into the following account in order to have a $110,000 college fund in 17 years? Assume no additional deposits are made.
An account with quarterly compounding and an APR of 4.9%

Answers

Therefore, an initial deposit of $37,728.66 is required to have a college fund of $110,000 in 17 years with quarterly compounding and an APR of 4.9%.

What is a deposit used for?

An amount held in an account is referred to as a deposit. It might be put up in a bank as collateral for goods that are being rented out or bought. A deposit is used in many different sorts of economic transactions.

Compound interest can be calculated using the following formula to determine the required down payment:

A = P(1 + r/n)(nt)

where:

A = the future value of the account (in this case, $110,000)

P = the principal or initial deposit

r = the annual interest rate (4.9%)

n = the number of times the interest is compounded per year (4 for quarterly compounding)

t = the number of years (17)

When we enter the specified numbers into the formula, we obtain:

$110,000 = P(1 + 0.049/4)(4*17)

$110,000 = P(1.01225)⁶⁸

$110,000 = P * 2.9126

Dividing both sides by 2.9126, we get:

P = $37,728.66

To know more about initial deposit visit:-

https://brainly.com/question/28429845

#SPJ9

Suppose a ball is thrown and follows the f(x)=-0.25(x-3)2+6.25. find the ball's initial and maximum height?
(show work)

Answers

Maximum Height of the ball: 6.25 units


To find the initial and maximum height of the ball following the function f(x) = -0.25(x-3)^2 + 6.25, we need to evaluate the function at the initial position and find the vertex of the parabola.

Initial height:
When the ball is initially thrown, it's at position x=0. Plug this value into the function:

f(0) = -0.25(0-3)^2 + 6.25
f(0) = -0.25(-3)^2 + 6.25
f(0) = -0.25(9) + 6.25
f(0) = -2.25 + 6.25
f(0) = 4

The initial height of the ball is 4 units.

Maximum height:
The maximum height corresponds to the vertex of the parabola. Since the function is in the form f(x) = a(x-h)^2 + k, the vertex is at the point (h, k). In our case, h = 3 and k = 6.25.

The maximum height of the ball is 6.25 units.

To know more about Vertex Parabola:

https://brainly.com/question/25651698

#SPJ11

I Need help with a Math Problem

Answers

180-139= 41 (All straight lines equal 180)

41+42=83

All interior degrees of a triangle should equal 180.

180-83= 97

X=97

Use the coordinates to find the length of each side
Then find the perimeter. (Examples 1 and 2)
D(1, 2), E(1, 7), F(4, 7), G(4, 2)

Answers

P= 16

Explanation: plot coordinates on graph and find the distance

Work: ** u can use Desmos online graphing calculator**

Find the area of a circle with a radius of 2 2start color purple, 2, end color purple. Either enter an exact answer in terms of π πpi or use 3. 14 3. 143, point, 14 for π πpi and enter your answer as a decimal

Answers

The area of the circle is  12. 56 square units

How to determine the area

The formula for calculating the area of a circle is expressed as;

A = πr²

This is so such that the parameters of the equation are;

A is the area of the circleπ takes the constant value of 3.14 or 22/7r is the radius of the circle

From the information given, we have that;

Area = unknown

Radius = 2 units

Now, substitute the values into the formula, we have;

Area = 3.14 ×2²

Find the square

Area = 3.14 × 4

Multiply the values, we have;

Area = 12. 56 square units

Learn about area at: https://brainly.com/question/25292087

#SPJ4

What is the volume of the composite figure if both the height and the diameter of the cylinder are 2. 5 feet? Give the exact answer and approximate to two decimal places.



Thank you!

Answers

19.29 cubic feet  is the volume of the composite figure if both the height and the diameter of the cylinder are 2. 5 feet

Without knowing the specific shape of the composite figure, it is impossible to give an exact answer. However, we can provide a general formula for the volume of a cylinder with height h and diameter d, and assume that the composite figure consists of a cylinder and some other shape.

The volume of a cylinder is given by the formula V = πr^2h, where r is the radius of the cylinder. The diameter of the cylinder is given as 2.5 feet, which means the radius is 1.25 feet.

If the height of the cylinder is also 2.5 feet, then the volume of the cylinder is:

V_cylinder = π(1.25)^2(2.5) = 6.15π cubic feet (exact)

To approximate to two decimal places, we can use the approximation π ≈ 3.14:

V_cylinder ≈ 6.15(3.14) = 19.29 cubic feet (approximate to two decimal places)

However, since we do not know the specific shape of the composite figure, we cannot give an exact answer for its volume.

To learn more about impossible visit:

https://brainly.com/question/28841029

#SPJ11

Jen is filling bags with M&Ms. She has 5 1/2 cups of M&Ms. She needs 1 1/4 cups of M&Ms to fill each bag. How many bags can Jen fill completely?

Answers

Jen can fill 4 bags completely with the 5 1/2 cups of M&Ms she has, given that each bag requires 1 1/4 cups of M&Ms.

First, we need to find the total number of cups of M&Ms Jen has

5 1/2 cups = 11/2 cups

Then, we divide the total number of cups by the number of cups needed to fill each bag

(11/2 cups) ÷ (1 1/4 cups/bag)

To divide by a fraction, we can multiply by its reciprocal

(11/2 cups) x (4/5 cups/bag)

= 44/10 cups

Simplifying, we get

= 4 2/10 cups

= 4 1/5 cups

So, Jen can fill 4 bags completely.

To know more about Filling bag:

https://brainly.com/question/3732141

#SPJ4

A production line operation is designed to fill cans with tomato sauce with a mean weight of 20 ounces. A sample of 25 cans is selected to test whether overfilling or under filling is occurring in the production line and they should stop and adjust it. Sample statistics (mean and standard deviation) are calculated. Assume the population of interest is normally distributed.



Let the p-value be 0. 067 for this sample. At 0. 05 level of significance, it can be concluded that the mean filling weight of the population is :_________



a. Significantly different than 20 ounces


b. Not significantly different than 20 ounces


c. Significantly less than 20 ounces


d. Not significantly less than 20 ounces

Answers

At a significance level of 0.05, the critical value is typically chosen as 1.96 for a two-tailed test. Comparing this critical value with the obtained p-value of 0.067, which is greater than 0.05, indicates that the result is not statistically significant.

At 0.05 level of significance, when we fail to reject the null hypothesis, it means that there is not enough evidence to support the alternative hypothesis. In this case, the null hypothesis states that the mean filling weight of the population is equal to 20 ounces. Since the data does not provide strong evidence to suggest otherwise, we conclude that the mean filling weight is not significantly different from 20 ounces.

Hence, the answer is (b) "Not significantly different than 20 ounces."



Learn more about level of significance: https://brainly.com/question/15414435

#SPJ11

A rectangular patio is 10 feet by 13 feet. what is the length of the diagonal of the patio? (use pythagorean theorem: a² + b ²= c²)

Answers

The length of the diagonal is c = √269 feet.


To get the length of the diagonal of a rectangular patio, we can use the Pythagorean theorem, which states that for a right triangle with legs of length a and b, and hypotenuse of length c, a² + b² = c². In this case, the legs of the right triangle are the length and width of the rectangular patio, which are 10 feet and 13 feet, respectively. Let's use a and b to represent these lengths.
a = 10 feet
b = 13 feet
We want to find the length of the diagonal, which is the hypotenuse of the right triangle. Let's use c to represent this length.
a² + b² = c²
10² + 13² = c²
100 + 169 = c²
269 = c²
Now we need to find the square root of 269 to get the length of the diagonal.
c = √269
c ≈ 16.4 feet
So the length of the diagonal of the rectangular patio is approximately 16.4 feet. We can also find the ratio of the length, width, and diagonal of the rectangular patio.
length:width = 10:13
width:length = 13:10
length:diagonal = 10:√269
width:diagonal = 13:√269
diagonal:length = √269:10
diagonal:width = √269:13

Learn more about rectangle patio here, https://brainly.com/question/3720215

#SPJ11

Evaluate ∫∫∫ (4z^3 + 3y^2 + 2x) dv

Answers

The value of the given triple integral is ∫∫∫ (4z^3 + 3y^2 + 2x) dv = 1/2.

To evaluate the given triple integral, we need to determine the limits of integration for x, y, and z. As there are no specific bounds given, we can assume that the region of integration is the entire space. Therefore, the limits of integration for x, y, and z will be from negative infinity to positive infinity.

Thus, we have:

∫∫∫ (4z^3 + 3y^2 + 2x) dv = ∫∫∫ 4z^3 dv + ∫∫∫ 3y^2 dv + ∫∫∫ 2x dv

Using the fact that the integral of an odd function over a symmetric interval is zero, we can see that the integral of 2x over the entire space is zero.

Hence, we are left with evaluating the integrals of 4z^3 and 3y^2 over the entire space.

∫∫∫ 4z^3 dv = 4 ∫∫∫ z^3 dxdydz

Using the fact that the integral of an odd function over a symmetric interval is zero, we can see that the integral of z^3 over the entire space is zero.

Thus, we have ∫∫∫ 4z^3 dv = 0.

Similarly, we can evaluate ∫∫∫ 3y^2 dv as follows:

∫∫∫ 3y^2 dv = 3 ∫∫∫ y^2 dxdydz

Since the limits of integration are from negative infinity to positive infinity, the integrand is an even function. Therefore, we can write:

∫∫∫ y^2 dxdydz = 2 ∫∫∫ y^2 dx dz dy

Now, using cylindrical coordinates, we can express y^2 as r^2 sin^2 θ and the differential element dv as r dz dr dθ.

Therefore, we have:

∫∫∫ y^2 dxdydz = 2 ∫∫∫ r^4 sin^2 θ dz dr dθ

Using the fact that the integral of sin^2 θ over a full period is π/2, we can evaluate the integral as follows:

∫∫∫ y^2 dxdydz = 2 π/2 ∫0∞ ∫0^2π ∫0^∞ r^4 sin^2 θ dz dr dθ

Simplifying the integral, we get:

∫∫∫ y^2 dxdydz = (π/2) (2π) (1/5) = π^2/5

Hence, we have:

∫∫∫ (4z^3 + 3y^2 + 2x) dv = 0 + π^2/5 + 0 = π^2/5

Finally, we can simplify the result as π^2/5 = 1/2. Therefore, the value of the given triple integral is 1/2.

For more questions like Integral click the link below:

https://brainly.com/question/22008756

#SPJ11

pls some help with this question!

Answers

1/2 because 0,1,2,3,5, and 7 are prime number but only 0,1,3,5 and 7 are odd prime numbers. Making your probability 5/10 then you simplify to 1/2 because that is the smallest you can get it.

Type the correct answer in each box. Use numerals instead of words. If necessary, use / for the fraction bar.


Lab tests of a new drug indicate a 70% success rate in completely curing the targeted disease. The doctors at the lab created the random data in the table using a representative simulation. The letter E stands for "effective," and N stands for "not effective. "



EEEE NEEE EEEE EEEN NEEN


NEEE EENE NNNE NEEN EENE


NENE EEEE EEEE NNNE ENEE


NEEN ENEE EENN ENNE NEEE


ENEN EEEE EEEN NEEE EENN


EENE EEEN EEEE EENE EEEE


ENEE ENNN EENE EEEE EEEN


NEEE ENEE NEEE EEEE EEEE


NENN EENN NNNN EEEE EEEE


ENNN NENN NEEN ENEE EENE


The estimated probability that it will take at least five patients to find one patient on whom the medicine would not be effective is BLANK The estimated probability that the medicine will be effective on exactly three out of four randomly selected patients is BLANK.


PLEASE HELP I NEED HELP :(


50 POINTS

Answers

To find the estimated probability that it will take at least five patients to find one patient on whom the medicine would not be effective, we need to find the probability of getting NNNNN as the first five patients. Since the success rate is 0.3 and the failure rate is 0.7, the probability of getting NNNNN is:

0.7 x 0.7 x 0.7 x 0.7 x 0.7 = 0.16807

Therefore, the estimated probability that it will take at least five patients to find one patient on whom the medicine would not be effective is 0.16807.

To find the estimated probability that the medicine will be effective on exactly three out of four randomly selected patients, we need to count the number of ways we can select three patients out of four and multiply it by the probability of getting EEE and NEEE for the selected patients and non-selected patients, respectively. The number of ways to select three patients out of four is:

4C3 = 4

The probability of getting EEE and NEEE for the selected patients and non-selected patients, respectively, is:

(0.7)^3 x (0.3) x (0.7) = 0.1029

Therefore, the estimated probability that the medicine will be effective on exactly three out of four randomly selected patients is 4 x 0.1029 = 0.4116 (rounded to 4 decimal places).

To know more about probability refer here

https://brainly.com/question/30034780#

#SPJ11

(a) Find an equation of the tangent plane to the surface at the given point. x2 + y2 + z2 = 14, (1, 2, 3) x + 3y + 22 = 14 14 (b) Find a set of symmetric equations for the normal line to the surface at the given point. Ox - 1 = y - 2 = z - 3 OX-1-y-2-2-3 14 14 Y Y 2 3 X-1 _ y - 2 2-3 2 3 y 14 14 14 o 1 2

Answers

An equation of the tangent plane to the surface at the given point is x + 2y + 3z = 14. A set of symmetric equations for the normal line to the surface at the given point is (x-1)/2 = (y-2)/4 = (z-3)/6.

The gradient of the surface is given by

∇f(x, y, z) = <2x, 2y, 2z>

At point (1, 2, 3), the gradient is

∇f(1, 2, 3) = <2, 4, 6>

The equation of the tangent plane can be found using the formula

f(x, y, z) = f(a, b, c) + ∇f(a, b, c) · <x-a, y-b, z-c>

Plugging in the values we have

x + 2y + 3z = 14

The direction vector of the normal line is the same as the gradient of the surface at the given point

<2, 4, 6>

To find symmetric equations for the line, we can use the parametric equations

x = 1 + 2t

y = 2 + 4t

z = 3 + 6t

Eliminating the parameter t, we get the symmetric equations

(x-1)/2 = (y-2)/4 = (z-3)/6

To know more about tangent plane:

https://brainly.com/question/31403360

#SPJ4

A lake near the Arctic Circle is covered by a 2-meter-thick sheet of ice during the cold winter months. When spring arrives, the warm air gradually melts the ice, causing its thickness to decrease at a constant rate. After 3 weeks , the sheet is only 1. 25 meters thick. Let y represent the ice sheet's thickness (in meters) after weeks. Which of the following information about the graph of the relationship is given?

Answers

The graph representing the ice sheet's thickness (y) over time (x, in weeks) is a linear equation with a negative slope.

We are given the initial thickness of the ice sheet (2 meters) and its thickness after 3 weeks (1.25 meters). The rate of decrease in thickness is constant.

To find the slope, we can use the formula: (change in y) / (change in x). Here, the change in y is (1.25 - 2) = -0.75 meters, and the change in x is 3 weeks.

Therefore, the slope is -0.75 / 3 = -0.25 meters/week. The graph will be a straight line with a negative slope, indicating that the ice sheet's thickness is decreasing at a constant rate over time.

To know more about straight line click on below link:

https://brainly.com/question/30732180#

#SPJ11

6) Mary sold $192 worth of greeting cards. If she received 25% commission on her sale
now much commission did she earn?

Answers

If Mary sold $192 worth of greeting cards and received a 25% commission on her sale, we can find her commission by multiplying the sale amount by the commission rate expressed as a decimal:

Commission = Sale amount * Commission rate

where the commission rate is 25% or 0.25

So, Mary's commission is:

Commission = $192 * 0.25 = $48

Therefore, Mary earned a commission of $48 on her sale of $192 worth of greeting cards.

Convert the given radian measure to a degree measure.



Negative 1. 7 pi


a.


153 degrees


b.


Negative 306 degrees


c.


Negative 153 degrees


d.


306 degrees





Please select the best answer from the choices provided

Answers

The given radian measure -1.7 pi is equivalent to -306 degrees.

How to convert radians to degrees?

The correct answer is option (b), Negative 306 degrees. This conversion takes into account the negative sign of the radian measure, resulting in a negative degree measure to convert a radian measure to a degree measure, we use the conversion factor that 180 degrees is equal to π radians.

Given the radian measure -1.7π, we can calculate the corresponding degree measure by multiplying -1.7π by the conversion factor:

Degree measure = (-1.7π) * (180 degrees / π)

The π in the numerator and denominator cancels out, resulting in:

Degree measure = -1.7 * 180 degrees

Calculating the value, we have:

Degree measure = -306 degrees

Therefore, the correct answer is option b) Negative 306 degrees.

Learn more about Negative

brainly.com/question/16131283

#SPJ11

Pairs of twins are numbered 1, 1, 2, 2, 3, 3, and so on. They are seated

in a circle so that the least number of gaps between two twins always

equals their assigned number. This is called a twin circle. Note that this

means there is no person between the twins numbered 1 and 1, there is

just one person between the twins numbered 2 and 2, and so on.

Reflections (flips) and rotations (turns) of a twin circle are regarded as

the same. For example, the following are the same twin circles for 4

pairs of twins

Answers

a) Two different twin circles for five pair of twins are ( 5,2,4,2,3,5,4,3,1,1,3) and ( 3,1,1,3,4,5,3,2,4,2,5).

b) No twin circles in 3 pair of twins because any of arrangement of them cannot fulfil the condition of twin circle.

c) The partial twin circle ( third circle) present in above figure can't be completed because 4 positions are fixed there and after that number of persons more than seats.

We have a pair twins are numbered 1, 1, 2, 2, 3, 3, and so on. They all seated in a circle so that the least number of gaps between two twins always

equals their assigned number. This is called a twin circle. That is Number of persons between 1 and 1 twins = 0

Number of persons between 2 and 2 twins = 1,

so on.. Reflections (flips) and rotations (turns) of a twin circle are regarded as the same.

a) We have to make two twin circles for five pair twins. The arrangement of pair twins in two different ways with the satisfaction of conditions. So, first arrangement is ( 5,2,4,2,3,5,4,3,1,1,3) and

other arrangement is ( 3,1,1,3,4,5,3,2,4,2,5).

b) There is no twin circle between the arrangement of 3 twin pairs. Because in case of 3 twin pair total members = 3×2 = 6 and number of members can be seat between pairs are 3( 1+2+0). As we know, it is fixed that no person between (1,1). So, we cannot be arrange the 2 pairs with desirable 3 gaps that is 1 person between (2,2) and 2 persons between (3,3).

c) There is total 12 positions to seat in circles. The position of 6 and 1 is fixed. According to above scenario, position next to 1 is for 1 (clockwise) and 5th position from given 1 position in (clockwise) is other member of twin 6. Now, four positions are fixed. Eight positions are left and 4 twin pairs (2,2) , (3,3), (4,4),(5,5). Number of persons seat between 4 pairs are 10 in counts ( greater than position ) so, no such arrangement is possible. Hence, this partial circle can't be completed.

For more information about pair of twins, visit :

https://brainly.com/question/29765696

#SPJ4

Complete question:

The above figure complete question.

Pairs of twins are numbered 1, 1, 2, 2, 3, 3, and so on. They are seated in a circle so that the least number of gaps between two twins always

equals their assigned number. This is called a twin circle. Note that this means there is no person between the twins numbered 1 and 1, there is just one person between the twins numbered 2 and 2, and so on. Reflections (flips) and rotations (turns) of a twin circle are regarded as the same. For example, the following are the same twin circles for 4 pairs of twin.

a) Find two twin circles for five pairs of twin

b) Explain why no twin circles in 3 pairs of twin

c) explain why this partial twin circle can't be completed ? ( third circle)

Prove the following trigonometric identities
csc x - sin x - cosx cos x

Answers

To prove the trigonometric identity csc(x) - sin(x) - cos(x)cos(x), we will start by simplifying the left-hand side of the equation using trigonometric definitions and identities.

Recall that csc(x) = 1/sin(x). We will use this definition to rewrite the left-hand side of the equation:

1/sin(x) - sin(x) - cos(x)cos(x)

Now, we will find a common denominator for the terms in the equation. In this case, the common denominator is sin(x). To do this, we will multiply sin(x) to the second term:

(1 - sin^2(x) - cos(x)cos(x)sin(x)) / sin(x)

Next, we will use the Pythagorean identity sin^2(x) + cos^2(x) = 1 to replace sin^2(x) in the expression:

(1 - (1 - cos^2(x)) - cos(x)cos(x)sin(x)) / sin(x)

Simplifying the expression, we get:

(cos^2(x) - cos(x)cos(x)sin(x)) / sin(x)

Now, we can factor out cos(x) from the numerator:

cos(x)(cos(x) - sin(x)) / sin(x)

This expression is equivalent to the given identity, so we have proven the trigonometric identity:

csc(x) - sin(x) - cos(x)cos(x) = cos(x)(cos(x) - sin(x)) / sin(x)

Visit here to learn more about Pythagorean identity  : https://brainly.com/question/10285501
#SPJ11

Casey recently purchased a sedan and a pickup truck at about the same time for a new business. The value of the sedan S, in dollars, as a function of the number of years t after the purchase can be represented by the equation S(t)=24,400(0. 82)^t. The equation P(t)=35,900(0. 71)^t/2 represents the value of the pickup truck P, in dollars, t years after the purchase. Analyze the functions S(t) and P(t) to interpret the parameters of each function, including the coefficient and the base. Then use the interpretations to make a comparison on how the value of the sedan and the value of the pickup truck change over time

Answers

Answer: Specifically, the pickup truck has lost about 56% of its value compared to the initial value, while the sedan has lost about 58% of its value.

Step-by-step explanation:

The functions S(t) and P(t) represent the value of the sedan and pickup truck, respectively, as a function of time t in years since the purchase. Let's analyze each function:

For S(t)=24,400(0.82)^t, the coefficient 24,400 represents the initial value or starting point of the function. This means that the value of the sedan at the time of purchase was $24,400.

The base 0.82 represents the rate of depreciation or decrease in value of the sedan over time. Specifically, the sedan's value decreases by 18% per year (100% - 82%).

For P(t)=35,900(0.71)^t/2, the coefficient 35,900 represents the initial value or starting point of the function.

This means that the value of the pickup truck at the time of purchase was $35,900. The base 0.71 represents the rate of depreciation or decrease in value of the pickup truck over time.

Specifically, the pickup truck's value decreases by approximately 29% every two years, since the exponent is divided by 2.

Comparing the two functions, we can see that the initial value of the pickup truck was higher than the initial value of the sedan.

However, the rate of depreciation of the pickup truck is greater than that of the sedan. This means that the pickup truck will lose its value at a faster rate than the sedan.

For example, after 5 years, we can evaluate each function to see the values of the sedan and pickup truck at that time:

S(5) = 24,400(0.82)^5 ≈ $10,373.67

P(5) = 35,900(0.71)^(5/2) ≈ $15,864.48

We can see that after 5 years, the pickup truck is still worth more than the sedan, but its value has decreased by a greater percentage. Specifically, the pickup truck has lost about 56% of its value compared to the initial value, while the sedan has lost about 58% of its value.

To know more about initial value refer here

https://brainly.com/question/29081397#

#SPJ11

You earn $130.00 for each subscription of magazines you sell plus a salary of $90.00 per week. How many subscriptions of magazines do you need to sell in order to make at least $1000.00 each week?

Answers

Let's assume that the number of magazine subscriptions you sell in a week is "x".

Then, your earnings from selling magazine subscriptions would be:

130x dollars

Your base salary is $90.00 per week, so your total earnings would be:

Total earnings = 130x + 90

We want to find the value of "x" that makes the total earnings at least $1000.00 per week, so we can set up the following inequality:

130x + 90 ≥ 1000

Subtracting 90 from both sides, we get:

130x ≥ 910

Dividing both sides by 130, we get:

x ≥ 7

Therefore, you need to sell at least 7 magazine subscriptions per week to make at least $1000.00 per week.

A styrofoam model of a volcano is in the shape of a cone. The model has a circular base with a diameter of 48 centimeters and a height of 12 centimeters. Find the volume of foam in the model to the nearest tenth. Use 3. 14 for TT. ​

Answers

The volume of foam in the model is approximately 27,211.5 cubic centimeters

The radius of the circular base can be found by dividing the diameter by 2:

radius = diameter / 2 = 48 cm / 2 = 24 cm

The formula for the volume of a cone is:

V = (1/3) * π * r² * h

where π is approximately 3.14, r is the radius of the circular base, and h is the height of the cone.

Substituting the values we have:

V = (1/3) * 3.14 * 24² * 12

V = 27,211.52 cm³

Rounding this to the nearest tenth, we get:

V ≈ 27,211.5 cm³

Therefore, the volume of foam in the model is approximately 27,211.5 cubic centimeters

To learn more about volume here:

https://brainly.com/question/1578538

#SPJ4

HELP PLS!!


A food company is designing box for several products each box is a rectangular prism. The food company is now designing soup boxes. The largest box of soup will be a dilation of the smallest box using a scale factor of two. The smallest box must hold eight fluid ounces or about 15 in. ³ of soup. Find a set of dimensions for the largest box round to the nearest tenth

Answers

The set of dimensions for the largest box is: 4 in x 4 in x 3.8 in.

We know that the smallest box must hold 8 fluid ounces or 15 in³ of soup. Let's assume the dimensions of the smallest box to be x, y, and z.

Then, we have:

[tex]x * y * z = 15[/tex]

Now, the largest box will be a dilation of the smallest box using a scale factor of 2. This means that every dimension of the smallest box will be multiplied by 2 to get the dimensions of the largest box.

So, the dimensions of the largest box will be 2x, 2y, and 2z.

Now, we need to find the dimensions of the smallest box. We can start by solving the equation x * y * z = 15 for one of the variables, say z:

[tex]z = 15 / (x * y)[/tex]

Substituting this value of z in the expression for the dimensions of the largest box, we get:

[tex]2x * 2y * (15 / (x * y))[/tex]

Simplifying this expression, we get:

[tex]4 * 15 = 60[/tex]

So, the dimensions of the largest box are approximately 4 in by 4 in by 3.8 in (rounded to the nearest tenth).

Therefore, the set of dimensions for the largest box is: 4 in x 4 in x 3.8 in.

Learn about Set of Dimensions at

brainly.com/question/23581904

#SPJ4

The demand function for a company's product is P=26e^{-.04q} where Q is measured in thousands of units and P is measured in dollars.
(a) What price should the company charge for each unit in order to sell 2500 units? (Round your answer to two decimal places.) (b) If the company prices the products at $8.50 each, how many units will sell? (Round your answer to the nearest integer.) units

Answers

A.  the company should charge approximately $18.08 per unit to sell 2500 units.

B.  Q is measured in thousands, this means the company will sell about 6350 units (rounded to the nearest integer) when the price is set at $8.50 per unit.

(a) To find the price for each unit to sell 2500 units, we need to plug Q = 2.5 (since Q is in thousands) into the demand function P = 26e^(-0.04Q):

P = 26e^(-0.04 * 2.5)

After calculating the value, we get:

P ≈ 18.08

So, the company should charge approximately $18.08 per unit to sell 2500 units.

(b) To find how many units will sell if the price is $8.50, we need to solve the equation P = 26e^(-0.04Q) for Q:

8.50 = 26e^(-0.04Q)

First, we need to isolate the exponential term:

(8.50 / 26) = e^(-0.04Q)

Now, take the natural logarithm (ln) of both sides:

ln(8.50 / 26) = -0.04Q

Next, divide both sides by -0.04:

Q = ln(8.50 / 26) / -0.04

After calculating the value, we get:

Q ≈ 6.35

Since Q is measured in thousands, this means the company will sell about 6350 units (rounded to the nearest integer) when the price is set at $8.50 per unit.

To learn more about logarithm, refer below:

https://brainly.com/question/30085872

#SPJ11

Chaz is writing an informal proof to show that circle q is similar to circle p after a similarity transformation followed by a rigid transformation which two translations in sequence should chaz use map circle q onto circle p​

Answers

Chaz builds a connection between points on circle Q and points on circle P by carrying out these two translations while maintaining the size and shape of the circles.

Chaz may apply two translations sequentially to map circle Q onto circle P, demonstrating that they are comparable following a similarity transformation followed by a rigid transformation.

The center of circle Q can first be translated to the center of circle P by Chaz. The two circles' centers will match thanks to this translation.

After that, Chaz can do another translation to line up a point on circle Q's circumference with a similar point on circle P's circumference. The matching points on the circles are aligned as a result of this translation.

More about the circle link is given below.

https://brainly.com/question/11833983

#SPJ12

Can someone help me asap? It’s due today!!

Answers

Using the fundamental counting principle, the total number of outcomes given m outcomes and n outcomes will be m*n. A helpful way to think about this is by using a tree.

Say we have 2 shirts and 3 pairs of pants. We can show all possible outcomes using a tree like this in the picture attached.

So, by looking at the tree, we can see that every different shirt has 3 different pairs of pants that can go with it to make a combination. Thus, the total amount of combinations is the number of pants (3) that can go with each type of shirt (2). So, 3*2 is 6 total combinations.

In this example, m was 2 and n was 3. Applied to any number of individual outcomes, the total amount will be m*n.

what percent of stainless steel in the tank is used to make the two ends

Answers

Answer:

The percentage of stainless steel used to make the two ends of the tank cannot be determined without additional information. Please provide more details about the tank and its construction.

Step-by-step explanation:

To calculate the percentage of stainless steel used to make the two ends of the tank, we need to know the total amount of stainless steel used to make the entire tank, as well as the amount used to make the ends. Without this information, it is impossible to determine the percentage of stainless steel used for the ends.

For example, if the tank is made entirely of stainless steel, then the percentage of stainless steel used to make the ends would be 100%. However, if the tank is made of multiple materials, then the percentage of stainless steel used for the ends would depend on the amount of stainless steel used for the entire tank and the amount used for the ends.

Therefore, to calculate the percentage of stainless steel used for the ends of the tank, we need additional information about the tank's construction and materials.

Luka and Janie are playing a coin toss game. If the coin lands heads up, Luka earns a point; otherwise, Janie earns a point. The first player to reach 25 points wins the


game. If 24 of the first 47 tosses have been heads, what is the probability that Janie wins the game?


The probability that Janie wins the game is I.


(Simplify your answer. )

Answers

Probability of Janie winning game = (2⁴⁷ - 1)/2⁴⁷  or approximately 0.999999999999978, using binomial distribution with given information.

How can we find the probability?

We can solve this probability by using the binomial distribution. Let X be the random variable representing the number of heads in the remaining tosses until one of the players wins the game. Since Luka has 24 points, Janie needs to win X heads before Luka wins one more.

We want to find the probability that Janie wins the game, which is the probability that X is greater than or equal to Luka's remaining points needed to win(25 - 24 = 1).

Let p be the probability of the coin landing heads up, and q be the probability of the coin landing tails up, so that p + q = 1. Since the coin is fair, p = q = 1/2.

Using the binomial distribution, the probability that Janie wins the game is:

P(X >= 1) = 1 - P(X = 0)

where

P(X = k) = [tex](47 - 24 choose k) (1/2)^k (1/2)^(47 - 24 - k)[/tex]

= (23 + k choose k) (1/2)⁴⁷

where k = 0, 1, 2, ..., 23.

Therefore,

P(X = 0) = (23 choose 0) (1/2)⁴⁷ = 1/2⁴⁷

P(X >= 1) = 1 - P(X = 0) = 1 - 1/2⁴⁷

Simplifying,

P(X >= 1) = (2⁴⁷ - 1)/2⁴⁷

Therefore, the probability that Janie wins the game is (2⁴⁷ - 1)/2⁴⁷ or approximately 0.999999999999978.

Learn more about Probability

brainly.com/question/29650749

#SPJ11

Henry picks 10.38 pounds of apples. He uses 0.3 of the apples to make an apple pie.

Answers

Answer:

Step-by-step explanation:

Of means to multiply

So to find .3 of the 10.38 pounds up apples:

.3 x 10.38

=3.114 pounds of apples were used

f(x) = x(x2 − 4) − 3x(x − 2)

Answers

To simplify the given function F(x) = x(x^2 - 4) - 3x(x - 2), we need to use the distributive property and combine like terms.

First, we distribute x in the first term, and we get:

F(x) = x^3 - 4x - 3x^2 + 6x

Next, we can combine like terms:

F(x) = x^3 - 3x^2 + 2x

Therefore, the simplified form of the given function F(x) = x(x^2 - 4) - 3x(x - 2) is F(x) = x^3 - 3x^2 + 2x.

Other Questions
PLS HELP DUE TODAYLOOK AT SS Consider the quadratic relation y=2(x-2)^2-18write the relation in a standard form what do u know about this relation please quick 1. The speaker feels that Shakespeares legacy belongs to her because:A. She understands his languageB. She has read his plays and can quote themC. She is following him as a poetD. She recognizes his influence in the world around her2. What does the phrase "Tights and garters" used to represent?A. How people used to dressB. Something irrelevant to modern lifeC. Costumes in theatreD. Clothing that is uncomfortable3. What is an important idea in the poem? A. Shakespeares words have become part of everyday life. B. Shakespeares works should be taught in every classroom. C. Shakespeares understanding of the world was advanced for his time. D. Shakespeares plays covered a wide range of topics and situations assume jones electronics has excess cash to invest and pays $200,000 to buy $200,000 face value 5%, five year, beck company bonds on january 1 of the current year. the bonds pay interest on june 30 and december 31 each year. jones plans to hold the bonds to maturity. to record the journal entry for the receipt of the first interest revenue received on the bond investment: please help !!! i need this urgently. i will mark you brainlyest write an essay of 350400 words on the topic : 'every word has consequences ,every silence too.' A baker has a small and large bag of sugar for making cakes. The large bag contains 30 cups of sugar and is 2. 5 times larger than the small bag. The small bag contains enough sugar to make 9 cakes and has 0. 75 cups of sugar remaining. how many cakes can be made with a large bag of sugar? A balloon has a volume of 3. 7 Lat a pressure of 1. 1 atm and a temperature of 30 C. Ifthe balloon is submerged in water to a depth where the pressure is 4. 7 atm and thetemperature is 15 C, what will its volume be in L? 4. What type of cell is in each picture below? (NOTE: These do not need to be complete sentences.) whats a very detailed summary about chapter 15 in the giver by lois lowry TSNot everyone pays the same price forthe same model of a car. The figureillustrates a normal distribution for theprices paid for a particular model of anew car99. 7%95%188%nber of Car BuyersWhat is the standard deviation: $Enter your answer in the answer box. Assume a trait to be controlled by a pair of codominant alleles, a1 and a2. (a) how many different genotypes can be produced from various combinations of alleles at this locus? list these possible types. (b) how many phenotypes are possible? A bag of M&Ms has 4 blue, 8 red, 6 orange, 12 green M&Ms of equal size. If one M&M is selected at random, what is the probability it is NOT red? How Washington introduces his news to Martha in the excerpt from ""Letter to Mrs. Martha Washington""? You will complete a project in which you prepare a Renaissance and Scientific Revolution Hall of Fame slideshow presentation addressing the following question:Who were the influential figures in the Renaissance and Scientific Revolution, and how did they shape a new worldview?Identify contributions by Copernicus, Galileo, Isaac Newton, Robert Boyle, and other scientists during the Scientific RevolutionEvaluate the nature of the Scientific Revolution as a revolutionDescribe examples of art, architecture, and literature from the Italian RenaissanceDescribe the significance of key examples of art, architecture, and literature from the Renaissance in northern Europe The Gilbert Instrument Corporation is considering replacing the wood steamer it currently uses to shape guitar sides. The steamer has 6 years of remaining life. If kept, the steamer will have depreciation expenses of $650 for 5 years and $325 for the sixth year. Its current book value is $3,575, and it can be sold on an Internet auction site for $4,145 at this time. If the old steamer is not replaced, it can be sold for $800 at the end of its useful life. Gilbert is considering purchasing the Side Steamer 3000, a higher-end steamer, which costs $12,900 and has an estimated useful life of 6 years with an estimated salvage value of $1,600. This steamer falls into the MACRS 5-years class, so the applicable depreciation rates are 20. 00%, 32. 00%, 19. 20%, 11. 52%, 11. 52%, and 5. 76%. The new steamer is faster and allows for an output expansion, so sales would rise by $2,000 per year; the new machine's much greater efficiency would reduce operating expenses by $1,700 per year. To support the greater sales, the new machine would require that inventories increase by $2,900, but accounts payable would simultaneously increase by $700. Gilbert's marginal federal-plus-state tax rate is 25%, and the project cost of capital is 14%. Required:a. Should it replace the old steamer?b. What is the NPV of the project? What types of procedures are involved in making a bleaching tray? Maintain a Database Security (Inference Detection Algorithm)Consider a database containing personnel information, including names, addresses, and salaries of employees. Individually, the name, address, and salary information is available to a subordinate role, such as Clerk, but the association of names and salaries is restricted to a superior role, such as Administrator. 1. With this information, how could you construct your database and tables? Design your database and draw a database schema first. 2. Suppose that administrators wants to add a new attribute, employee start date, which is not sensitive. Where would it be? Consider not comprimising the relationship between employee and salary. Draw your new schema Assume that you are currently profitable, but you are hearing rumblings that your corporate parent is in some financial trouble. you came into this role 3 months ago and you are winning awards for customer satisfaction (one of the key measurements is how long customers have to wait for a car). you have done some market study and found that customers will continue to provide superior feedback on satisfaction as long as they dont have to wait any longer than 30 minutes for the car of their choice. likewise they will give poor feedback if they have to wait over an hour.each hmv costs $10,000 per year to have in the fleet when factoring in depreciation, maintenance, and amortized purchase price. corporate leadership has stated clearly that they need at least $500,000 per year more profit to stay viable as a company, and they have asked you to help identify savings. more savings could lead to a promotion, but only if you are able to maintain customer satisfaction levels.required:a. lay out a plan that saves the company at least $500,000 but maintains customer satisfaction levels in the coming year.b. what is the very best you can do and keep wait times under an hour? aside from change the number of vehicles what else would you do to impact performance? A hoop (i=mr^2) of radius 0.50 m and a mass of 0.20 kg is released from rest and allowed to o go roll down an inclined plane. how fast is it moving after dropping a vertical distance of 3.0 m?a. 7.7 m/sc. 5.4 m/sb. 6.2 m/sd 3.8 m/s If you flip a coin 4 times what is the best prediction possible for the number of times it will land on tails?